Trouble with Integral to find area using Green's Theorem

Click For Summary
The discussion revolves around calculating the area of a region defined by the hypocycloid equation using Green's Theorem. The integral set up is \(\int^{2\pi}_{0}(3a^{2}\sin^{2}t\cos^{4}t)dt\), but the user struggles with evaluating it by hand. Suggestions are made to simplify the integrand using trigonometric identities, specifically rewriting \(\sin^{2}t\cos^{4}t\) for easier integration. The conversation highlights the challenge of the integral while exploring potential methods to solve it. Ultimately, the focus is on finding a manageable approach to compute the area accurately.
Opus_723
Messages
175
Reaction score
3

Homework Statement



Calculate the area of the region within the hypocycloid x^{2/3}+y^{2/3}=a^{2/3} parameterized by x=acos^{3}t, y=asin^{3}t, 0\leqt\leq2\pi

Homework Equations



In the problem prior to this one, I showed that the line integral of \vec{F}=x\hat{j} around a closed curve in the xy-plane, oriented as in Green's Theorem, measures the area of the region enclosed by the curve. I am supposed to use this fact to find the area above.

The Attempt at a Solution



So I've got it all set up right, but I'm having trouble with the integral. Either I'm just forgetting how to solve this sort of integral, or there's another, simpler way to do the problem. The integral I got is:

\int^{2\pi}_{0}(3a^{2}sin^{2}tcos^{4}t)dt

I checked it numerically on wolfram and this integral gives the right answer, but I don't know how to compute it by hand. Normally our prof doesn't assign anything with tricky integrals, so either this is easier than I'm thinking, or there's another way to find the area. But since this vector field isn't path-independent, I don't see any shortcuts I could take.
 
Physics news on Phys.org
You might try starting by writing everything in terms of sin t.
 
Well, I don't remember too many trig identities, but I'll give it a try.

Starting with sin^{2}t*cos^{4}t (Pulling 3a^{2} out as a constant)

cos^{2}t = 1-sin^{2}t

cos^{4}t = (cos^{2}t)^{2} = (1-sin^{2}t)^{2} = 1-2sin^{2}t+sin^{4}t

sin^{2}t*cos^{4}t = sin^{2}t-2sin^{4}t+sin^{6}t

Does that look right? The integral still doesn't jump out at me.
 
That's doable but tedious. Try rewriting the integrand like this:
$$\sin^2 t\ \cos^4 t = (\sin^2 t\ \cos^2 t)\cos^2 t = \frac{1}{4}\sin^2 2t\left(\frac{1+\cos 2t}{2}\right).$$ That's pretty straightforward to integrate after you multiply it out.
 
Question: A clock's minute hand has length 4 and its hour hand has length 3. What is the distance between the tips at the moment when it is increasing most rapidly?(Putnam Exam Question) Answer: Making assumption that both the hands moves at constant angular velocities, the answer is ## \sqrt{7} .## But don't you think this assumption is somewhat doubtful and wrong?

Similar threads

Replies
3
Views
2K
  • · Replies 10 ·
Replies
10
Views
2K
  • · Replies 7 ·
Replies
7
Views
2K
  • · Replies 3 ·
Replies
3
Views
2K
Replies
12
Views
2K
Replies
2
Views
2K
  • · Replies 2 ·
Replies
2
Views
2K
  • · Replies 3 ·
Replies
3
Views
2K
Replies
2
Views
1K
  • · Replies 5 ·
Replies
5
Views
1K